Volume for a cone in cylindrical coordinates.

Click For Summary
The discussion centers on calculating the volume of a cone using cylindrical coordinates, where the user struggles with setting the correct limits for integration. The integral setup appears correct, but the user mistakenly considers a 45° cone instead of a general cone, leading to confusion in the final formula. It is clarified that the integration limits need adjustment to yield the correct volume formula, V_cone = (1/3)πr²h. The importance of variable naming and the implications of using specific angles in the calculations are also noted. The conversation emphasizes the need for careful attention to integration limits and variable definitions in calculus.
Telemachus
Messages
820
Reaction score
30

Homework Statement


Hi there. I haven't used iterated integrals for a while, and I'm studying some mechanics, the inertia tensor, etc. so I need to use some calculus. And I'm having some trouble with it.

I was trying to find the volume of a cone, and then I've found lots of trouble with such a simple problem.

So I thought of using cylindrical coordinates this way:
\begin{Bmatrix}{ x=r\cos\theta} \\y=r\sin\theta \\z=r\end{matrix}

And then I've stated the integral this way:

\displaystyle\int_{0}^{2\pi}\displaystyle\int_{0}^{r}\displaystyle\int_{r}^{h}rdzdrd\theta=\displaystyle\int_{0}^{2\pi}\displaystyle\int_{0}^{r}r(h-r)drd\theta=\displaystyle\int_{0}^{2\pi}\displaystyle\frac{r^2h}{2}-\displaystyle\frac{r^3}{3}=\pi r^2h-\displaystyle\frac{2\pi\r^3}{3}=\pi r^2(h-\displaystyle\frac{2}{3}r)

But I should get: V_{cone}=\displaystyle\frac{\pi r^2 h}{3}

I think I'm giving wrong limits for the integration.

Help pls :)
 
Last edited:
Physics news on Phys.org
Hi Telemachus! :smile:
Telemachus said:
I think I'm giving wrong limits for the integration.

No, your integration is fine. :smile:

(I'd have used spherical coordinates, but your way does work)

But you're doing it for the 45° cone instead of a general cone (so r = h, which makes your formula the same as the given answer).

Somehow your variable of integration r has managed to survive into the afterlife under a new persona. :wink:
 
Thanks Tim :)

Haha sorry for the notation, I should used another name for the variable :P
 
Question: A clock's minute hand has length 4 and its hour hand has length 3. What is the distance between the tips at the moment when it is increasing most rapidly?(Putnam Exam Question) Answer: Making assumption that both the hands moves at constant angular velocities, the answer is ## \sqrt{7} .## But don't you think this assumption is somewhat doubtful and wrong?

Similar threads

  • · Replies 2 ·
Replies
2
Views
2K
  • · Replies 19 ·
Replies
19
Views
2K
Replies
3
Views
2K
  • · Replies 34 ·
2
Replies
34
Views
3K
  • · Replies 9 ·
Replies
9
Views
1K
  • · Replies 9 ·
Replies
9
Views
2K
Replies
9
Views
4K
Replies
12
Views
2K
Replies
3
Views
2K
Replies
14
Views
2K